Hatcher dump
by yayups, Sep 27, 2020, 8:15 PM
Section 1.1
As a general convention,
will be the path variable and
will be the homotopy variable.
Solution
Solution
Solution
Solution
Solution
Solution
Solution
As a general convention,


1.1.1 wrote:
Show that the composition of paths satisfies the following cancellation property: If
and
, then
.



Trivial by group properties.
1.1.2 wrote:
Show that the change-of-basepoint homomorphism
depends only on the homotopy class of
.


Suppose we have a path
from
to
, so we have the map
This only depends on
, so we're done.



![\begin{align*} \beta_h:\pi_1(X,x_1)&\to \pi_1(X,x_0) \\ [f]&\mapsto [h\cdot f\cdot \overline{h}] = [h]\cdot[f]\cdot[h]^{-1}. \end{align*}](http://latex.artofproblemsolving.com/4/7/4/4747e0b0715c43482201733e2800fb679404f357.png)
![$[h]$](http://latex.artofproblemsolving.com/5/5/8/5584515cc49cb2b664ff811b0d429147eb37f5df.png)
1.1.3 wrote:
For a path connected space
, show that
is abelian iff all basepoint-change homomorphisms depend only on the endpoints of the path
.



As before, suppose we have a path
from
to
, so we have the map
Fix any paths
from
to
and
from
to
, and let
. Note that
and since we can attain any
this way, we're done.



![\begin{align*} \beta_h:\pi_1(X,x_1)&\to \pi_1(X,x_0) \\ [f]&\mapsto [h]\cdot[f]\cdot[h]^{-1}. \end{align*}](http://latex.artofproblemsolving.com/d/6/2/d62f57d28d8ca576663e71e544eeebdef8a8b997.png)







![\begin{align*} \beta_{\bar{h_1}} = \beta_{h_2} &\iff [h_1]^{-1}\cdot[f]\cdot[h_1] = [h_2]^{-1}\cdot[f]\cdot[h_2]\quad\forall f\in\pi_1(X,x_1) \\ &\iff [f]\cdot [g] = [g]\cdot [f], \end{align*}](http://latex.artofproblemsolving.com/f/5/6/f56227e4d15f571660dd985a73c49f38cb80e58f.png)

1.1.4 wrote:
A subspace
is said to be star-shaped if there is a point
such that, for each
, the line segment from
to
lies in
. Show that if a subspace
is locally star-shaped, in the sense that every point of
has a star-shaped neighborhood in
, then every path in
is homotopic to a piecewise linear path, that is, a path consisting of a finite number of straight line segments traversed at constant speed. Show this applies in particular when
is open or when
is a union of finitely many closed convex sets.












We first prove the problem in the case where
is itself star-shaped.
Lemma: Suppose
is star-shaped. Then, any path
is homotopic to a piecewise linear path with at most
pieces. Note that in this homotopy of paths, the endpoints remain fixed.
Proof: First, scale the path so it is from
. Also WLOG let
be the center of the star of
. Now, consider the homotopy
Note this stays in
since it is a scaling of points in
by a factor
. This takes our original
to a composition of two paths, one from some point to
, and one from
to some other point. Thus, it suffices to show that if
is a path from
to some point, then it is homotopic to a linear path.
Indeed, suppose we have
such that
. Consider the homotopy
(which is a homotopy for the same reason) which sends
to a linear path from
to
. This completes the proof of the lemma. 
The finish is a standard compactness argument which I will write out in detail for practice. Suppose we have our path
where
is locally star-shaped. For each
, let
denote the open neighborhood of
that is star-shaped. We see that
is an open set in
, so for each
, pick
an open interval that contains
and is contained within
.
We see that the
s cover
, so by compactness of
, there is some finite collection
of them such that the
cover
. Between any two consecutive
s, pick a
that is between them and contained in
. Then, including all the half integer
s and re-indexing, we have
such that
is fully contained in some
. Thus, within
, we use the lemma to fix the part of
in
to piecewise linear while fixing the endpoints, so we have a homotopy from
to a piecewise linear path of at most
pieces.

Lemma: Suppose



Proof: First, scale the path so it is from
![$[-1,1]\to U$](http://latex.artofproblemsolving.com/9/5/8/9583814894ff6a632453e34f20c50fa241f08f87.png)


![\[F(s,t) = [1-s(1-|t|)]\cdot g(t).\]](http://latex.artofproblemsolving.com/e/b/1/eb1d3c0917fe9722394ec9560c703352a44c0a5a.png)








Indeed, suppose we have
![\[g:I\to U\]](http://latex.artofproblemsolving.com/e/f/8/ef87fb8aed5c13ae5bd14e49c5c978135965d7a6.png)

![\[F(s,t) = t\cdot g(s+(1-s)t),\]](http://latex.artofproblemsolving.com/d/d/b/ddb2827345060efad0efd136b65d087f0b121f94.png)




The finish is a standard compactness argument which I will write out in detail for practice. Suppose we have our path
![\[f:I\to X,\]](http://latex.artofproblemsolving.com/2/2/7/227fbfbb6ab5e9d445902e771fa4f01e45f0be4e.png)










We see that the










![\[0=t_0<t_1<t_2<\cdots<t_{m'}=1,\]](http://latex.artofproblemsolving.com/3/6/7/367d9537badc5068d533c5cf884fe853df18638e.png)
![$[t_i,t_{i+1}]$](http://latex.artofproblemsolving.com/9/a/b/9aba5be221b8c6726708a61dabf49e970e6978f1.png)



![$[t_i,t_{i+1}]$](http://latex.artofproblemsolving.com/9/a/b/9aba5be221b8c6726708a61dabf49e970e6978f1.png)


1.1.5 wrote:
Show that for a space
, the following three conditions are equivalent:
is simply connected iff all maps
are homotopic.

- Every map
is homotopic to a constant map, with image a point.
- Every map
extends to a map
.
for all
.


We may assume that
is path connected, since any map
lands in only one component of
, as
has a single component. We'll view
as the unit circle in
. Fix
. For all
, let
be a path from
to
.
For any map
, define
in the following way:
where
is the loop given by
.
For any loop
, define
by
. Since
, we see that this map is continuous (one can prove this formally by noting that
is continuous where
is the quotient map).
We'll first show (a)
(c). Suppose we have
. Let
be a homotopy such that
and
(constant). Now, define
by
We see that this a homotopy from
to
, which is the identity loop. Thus, every element of
is homotopic to the identity, so
.
We'll now show (c)
(a). Suppose we have
, which we want to show is homotopic to a constant map. Indeed, consider the homotopy
with
and
. We then have the homotopy
given by
This is a homotopy from
to a constant loop at
, so
is homotopic to a constant loop.
Finally, we'll show (a)
(b). The key is that
is homeomorphic to
where
is identified to a point, and furthermore, this homeomorphism identically maps
to the boundary of
. Now, we have a map
such that
for a fixed map
if and only if we have a continuous map
such that
and
is constant. This is exactly equivalent to saying that
is homotopic to a constant loop, so we're done! This shows all the desired equivalences.











For any map


![\[[\tilde{f}] = [\eta_{f(1)}]\cdot[g]\cdot[\eta_{f(1)}],\]](http://latex.artofproblemsolving.com/7/1/7/717be21cf5f988b83b241743129940e14fbf9c9d.png)


For any loop






We'll first show (a)






![\[\tilde{F}(t,s) = \widetilde{F(\bullet,s)}(t).\]](http://latex.artofproblemsolving.com/0/e/2/0e29b63964a7144648ad77fe157c1a67333abdee.png)

![$[\eta_{x_1}]\cdot[\eta_{x_1}]^{-1}$](http://latex.artofproblemsolving.com/6/9/3/693f7ba6b948d0364cb25b3dfe0a1281e761ba22.png)


We'll now show (c)






![\[\tilde{F}(\omega,s) = \widetilde{F(\bullet,s)}(\omega)\]](http://latex.artofproblemsolving.com/1/3/0/13096561d44b117daf8dfc9b50c5f717437dfc2a.png)



Finally, we'll show (a)






![\[F:S^1\times I/\sim\,\,\to X\]](http://latex.artofproblemsolving.com/0/1/d/01dc8001c9fa32e7d8304db406fadd08fc82ca28.png)


![\[\tilde{F}:S^1\times I\to X\]](http://latex.artofproblemsolving.com/7/0/9/709f0ba9ad799e29fd4679f282e97bace67fbd27.png)



1.1.6 wrote:
We can regard
as the set of basepoint-preserving homotopy classes of maps
. Let
be the set of homotopy classes of maps
with no conditions on basepoints. Thus there is a natural map
obtained by ignoring basepoints. Show that
is onto if
is path connected, and that
iff
and
are conjugate in
. Hence
induces a one-to-one correspondence between
and the set of conjugacy classes in
, when
is path connected.


![$[S^1,X]$](http://latex.artofproblemsolving.com/c/1/1/c11e4e8e083dbe4a02d442d3e41f63e74e5e3fdf.png)

![$\Phi:\pi_1(X,x_0)\to [S^1,X]$](http://latex.artofproblemsolving.com/5/0/0/500056a0ab5daca828eda0bfbe701170c74eb16a.png)


![$\Phi([f])=\Phi([g])$](http://latex.artofproblemsolving.com/2/a/3/2a3aef364a8d2ce1e27513617b70bab92925a3ff.png)
![$[f]$](http://latex.artofproblemsolving.com/f/4/4/f44bfb37b68791e7968284f5b7237bb59749656a.png)
![$[g]$](http://latex.artofproblemsolving.com/f/e/6/fe6f53448f516922ffc8265065784c6aa7db8525.png)


![$[S^1,X]$](http://latex.artofproblemsolving.com/c/1/1/c11e4e8e083dbe4a02d442d3e41f63e74e5e3fdf.png)


Keep the
and
notation from the previous solution. For any
, it is easy to see that
is homotopic to
where
is a path from
to
, so
is onto.
For one direction, it suffices to show that
for all
, and this is indeed true as we can compose
with a rotation to get
.
For the other direction, suppose that
. Let the image of the basepoint
under the homotopy
be
. Let the path from
to
along
(so the image of
for some
) be
, and the rest of
be
, so
. However, if
is a path from
to
, then the homotopy tells us that
It is not hard to check that this rearranges to
and since
, we see that
and
are conjugate in
, as desired.



![$\Phi([a]\cdot[\tilde{f}]\cdot[a]^{-1})$](http://latex.artofproblemsolving.com/b/a/7/ba7d0501fc86af993ad0d629057ad5f5227d7c96.png)





For one direction, it suffices to show that
![$\Phi([f]\cdot [g]) = \Phi([g]\cdot [f])$](http://latex.artofproblemsolving.com/4/c/4/4c44b623825d730a4bab5fbe921071c778bbedb6.png)

![$\Phi([f]\cdot[g])$](http://latex.artofproblemsolving.com/f/6/b/f6bf1cbb3bcff3bb71af57deb4bf9134dac67a28.png)
![$\Phi([g]\cdot[f])$](http://latex.artofproblemsolving.com/6/2/9/62996a727e65b2c6fb3b0ad43c9ee7b28e25c2e1.png)
For the other direction, suppose that
![$\Phi([f])=\Phi([g])$](http://latex.artofproblemsolving.com/2/a/3/2a3aef364a8d2ce1e27513617b70bab92925a3ff.png)

![$\Phi([f])\to\Phi([g])$](http://latex.artofproblemsolving.com/9/c/f/9cfcc27492cf550020f60f14cab830f1686d8a59.png)




![$g([0,t])$](http://latex.artofproblemsolving.com/3/5/5/3556ba065dabc6d1e47a98716ae36d835b51e932.png)




![$[g]=[g_2]\cdot[g_1]$](http://latex.artofproblemsolving.com/e/3/c/e3cba19e3466b3db94314bbc887d0cadc8f2bc4b.png)



![\[[a]\cdot[g_1]\cdot[g_2]\cdot[a]^{-1} = [f].\]](http://latex.artofproblemsolving.com/8/1/8/8181ee80c51213d2453b40acd190a6030dfcb519.png)
![\[[g] = ([a]\cdot[g_1])^{-1}\cdot[f]\cdot([a]\cdot[g_1]),\]](http://latex.artofproblemsolving.com/a/8/8/a88c060d9a54353a1a5f1c73214892d4f6f326a2.png)
![$[a]\cdot[g_1]\in\pi_1(X,x_0)$](http://latex.artofproblemsolving.com/e/9/8/e985e079ccbbe1ad18f2ea53a0ba81c28a7a0c65.png)
![$[g]$](http://latex.artofproblemsolving.com/f/e/6/fe6f53448f516922ffc8265065784c6aa7db8525.png)
![$[f]$](http://latex.artofproblemsolving.com/f/4/4/f44bfb37b68791e7968284f5b7237bb59749656a.png)

1.1.7 wrote:
Define
by
, so
restricts to the identity on the two boundary circles of
. Show that
is homotopic to the identity by a homotopy that is stationary on one of the boundary circles, but not by any homotopy that is stationary on both.





The homotopy
works just fine.
Suppose we had a homotopy
(here
is the homotopy variable) that sent
to the identity, while being stationary on both boundary circles. The key is to now consider what this homotopy does to the induced map
on the quotient
given by gluing the two boundary circles together. Since
is stationary on the boundary circles, it is a continuous map from
.
Now, for fixed
, and fixed
,
is a loop in
, and by varying
, we get a homotopy from this loop to the identity loop. If we project this loop and the homotopy onto the
that was not formed during the quotient process, we see that we have a have a homotopy between a loop with winding number
and the identity loop, which is a contradiction by
. Thus,
cannot exist.

Suppose we had a homotopy







Now, for fixed









This post has been edited 2 times. Last edited by yayups, Sep 27, 2020, 8:18 PM
Animation!
by yayups, Nov 15, 2019, 9:44 PM
Diffusion Problem
by yayups, Aug 7, 2019, 8:52 PM
In this post, we'll be solving the following very instructive physics problem.
A vessel with a small hole of diameter
is place inside a chamber much larger than it that is filled with a gas of temperature
and pressure
. The pressure
is very low, so in particular, the mean free path
of the gas is much larger than
. The vessel is kept a constant temperature
. Once steady state is reached, what is the pressure in the vessel?
Since the chamber is much larger than the vessel, gas entering the vessel and leaving from the vessel won't affect the pressure or temperature of the chamber.
At this point, it is very easy to fall into the following trap. One may argue that steady state is reached when the pressures in the vessel and chamber are the same. After all, isn't that what happens when two gases come in equilibrium?
The issue with this reasoning is due to the fact that we are in the regime where
. What this means is that for particles near the hole that have a chance of passing through it, there are unlikely to be any collisions. The effect of pressure arises through particle collisions, so in this case, particles will pass through the hole not due to a pressure difference, but rather through the random chance that their velocity has an appropriate magnitude and direction to go through the hole (this is diffusion). Steady state is reached when the rate of particles going from the chamber to the vessel is the same as the rate of particles going from the vessel to the chamber.
Thus, to solve the problem, we need to calculate the rate that particles leave through a hole in a box of gas, given that the size of the hole is much smaller than the mean free path.
Proof
The finish from here is easy. Suppose the particle number density in the chamber is
and in the vessel is
. Steady state implies the rate of particles flowing through the hole is the same from both sides, or that
The ideal gas law tells us that
, so
and
.
Some Graduate Entrance Exam Problem wrote:








Since the chamber is much larger than the vessel, gas entering the vessel and leaving from the vessel won't affect the pressure or temperature of the chamber.
At this point, it is very easy to fall into the following trap. One may argue that steady state is reached when the pressures in the vessel and chamber are the same. After all, isn't that what happens when two gases come in equilibrium?
The issue with this reasoning is due to the fact that we are in the regime where

Thus, to solve the problem, we need to calculate the rate that particles leave through a hole in a box of gas, given that the size of the hole is much smaller than the mean free path.
Lemma wrote:
Given a box with an (ideal) gas of particle mass
, temperature
, and number density
(number of particles per unit volume), the number of particles leaving a small hole of area
per unit time is
assuming that
.




![\[\frac{\eta A}{4}\langle v\rangle=\frac{\eta A}{4}\sqrt{\frac{8k_BT}{\pi m}}\]](http://latex.artofproblemsolving.com/8/8/6/886cf171b31e20a20b310f5185d3b7eaab1795dd.png)

Let
be the Maxwell-Boltzmann distribution of velocities of the particles. What this means is that the probability that a particle has velocity in
is
Set up spherical coordinates with origin at the hole. We will now count the number of particles that hit the hole in a time
using a funny double counting argument, where we start by counting the number of particles that hit the hole with a certain velocity and then integrate over all velocities.
We will start by counting the number of particles that move with speed
(technically speed in
, but from now on we'll be lazy about this) and spherical coordinate angles
. Here
means pointing toward the hole, and
is parallel to the plane of the hole (the spherical coordinates for the velocity are flipped compared to those for space, since the
rays are anti-parallel). In a given volume
, the number of particles with this velocity is just
For this given velocity, the volume in space that will allow such particles to hit the hole is a tilted cone object with base
, slant
, slant height
, and aligned in the proper
direction. In particular, its volume is
, so the number of particles with velocity
hitting the hole in time
is
Thus, the rate of particles leaving is
Note that
which tells us that
as desired. Note that proof didn't depend on the particular form of the Maxwell-Boltzmann speed distribution.

![$[v_x,v_x+dv_x]\times[v_y,v_y+dv_y]\times[v_z,v_z+dv_z]$](http://latex.artofproblemsolving.com/2/3/4/2347751741e1dbe9846903d39eae4b040d7275a4.png)
![\[f\left(\sqrt{v_x^2+v_y^2+v_z^2}\right)dv_xdv_ydv_z.\]](http://latex.artofproblemsolving.com/e/d/9/ed939a866b843f7a218b9fc1c8ff467f933d8332.png)

We will start by counting the number of particles that move with speed

![$[v,v+dv]$](http://latex.artofproblemsolving.com/e/3/6/e36557a3f0a889236a1e6388bc09705240ccb880.png)





![\[(\eta dV)\cdot f(v)\cdot v^2\sin\theta \,dv \,d\theta\, d\phi.\]](http://latex.artofproblemsolving.com/8/9/9/89937eabed07f31a25ce99d3d8736a99df1e7688.png)







![\[(\eta A dt)\cdot f(v)\cdot v^3\sin\theta\cos\theta \,dv \,d\theta\, d\phi.\]](http://latex.artofproblemsolving.com/8/4/3/843724208a7990c3e3524499b8b444cf5a48aefc.png)
![\[\alpha=\eta A\int_0^\infty v^3f(v)\,dv\int_0^{\pi/2}\sin\theta\cos\theta\,d\theta\int_0^{2\pi}d\phi=\pi\eta A\int_0^\infty v^3f(v)\,dv.\]](http://latex.artofproblemsolving.com/2/c/6/2c62a0f3e36a3c0148d8dd4ba9dd2de33fa2026d.png)
![\[\langle v\rangle=\int_0^\infty\int_0^\pi\int_0^{2\pi} v\cdot f(v)\cdot v^2\sin\theta\,dv\,d\theta\,d\phi=4\pi\int_0^\infty v^3f(v)\,dv,\]](http://latex.artofproblemsolving.com/c/8/c/c8cb9af6a1e632d0586359560e0b24ccaf0f11cb.png)
![\[\alpha=\frac{\eta A}{4}\langle v\rangle,\]](http://latex.artofproblemsolving.com/f/e/2/fe26c6f89cd559d3461d875d9628e80638cdf505.png)
The finish from here is easy. Suppose the particle number density in the chamber is


![\[\eta_0\langle v_0\rangle = \eta_1\langle v_1\rangle\implies \eta_0\sqrt{T_0}=\eta_1\sqrt{T_1}\implies \eta_0=2\eta_1.\]](http://latex.artofproblemsolving.com/5/5/3/553ae696d12775b53e1e0baa37fd4f7889e4f04e.png)



Paralogic Triangles
by yayups, Jun 13, 2019, 4:00 AM
In this post, we consider Paralogic Triangles. The content of this post comes from a discussion with tastymath75025 and pinetree1. It's also mostly from Lemmas in Olympiad Geometry.
![[asy]
unitsize(1.5inches);
pair A,B,C,D,E,F,X,Y,Z,P,Q,H,HH,R;
A=dir(105);
B=dir(210);
C=dir(330);
F=(0.3)*A+(0.7)*B;
E=(3/5)*A+(2/5)*C;
D=extension(E,F,B,C);
X=2*circumcenter(A,E,F)-A;
Y=2*circumcenter(B,D,F)-B;
Z=2*circumcenter(C,D,E)-C;
P=extension(A,X,B,Y);
Q=2*foot(P,circumcenter(A,B,C),circumcenter(X,Y,Z))-P;
H=orthocenter(A,B,C);
HH=orthocenter(X,Y,Z);
R=2*foot(Q,D,E)-Q;
draw(A--B--C--cycle,linewidth(1.5)+red);
draw(D--B);
draw(X--Y--Z--cycle,linewidth(1.5)+orange);
draw(X--E,dotted);
draw(circumcircle(A,B,C));
draw(D--F--E,blue);
draw(A--X--P,green);
draw(P--B--Y,green);
draw(Z--P--C,green);
draw(circumcircle(X,Y,Z));
dot("$A$",A,dir(A));
dot("$B$",B,dir(230));
dot("$C$",C,dir(C));
dot("$F$",F,1.5*dir(105));
dot("$E$",E,dir(0));
dot("$D$",D,dir(180));
dot("$X$",X,dir(-10));
dot("$Y$",Y,dir(160));
dot("$Z$",Z,dir(200));
dot("$P$",P,1.3*dir(270));
dot("$Q$",Q,1.3*dir(110));
dot("$H$",H,1.3*dir(0));
dot("$H'$",HH,1.3*dir(180));
dot("$R$",R,1.3*dir(-30));
[/asy]](//latex.artofproblemsolving.com/8/4/2/84257186b6f8aa72f72176e4e9c53b6fadd2c304.png)
Consider
, and let
be an arbitrary line that intersects the sides at
,
and
. Let
be the the perpendicular to
through
, and define
and
similarly. Finally, let
be the triangle determined by
,
and
.
It's clear that
because the repsective sides are perpendicular. In fact, the triangles are perspective through
which lies on
. Firstly, by Desargues's theorem, the triangles are perspective. To show that
, note that
since
cyclic, so
. We also see that
since the setup is symmetric under
interchange.
It turns out that
and
are in fact orthogonal. The key is to constuct the point
which is the other intersection of the two circles. Note that
and
, so
is the spiral center sending
. Thus,
is the center of the unique spiral similarity sending
. Now we see that any such spiral similarity must have angle equal to
since the corresponding sides of
and
are perpendicular. Thus, the unique spiral similarity sending
to
has angle
, which implies the circles are orthogonal.
The point
turns out to be quite nice, and in fact it is the Miquel point of
and
. To see this, note that by the spiral similarity,
, so
is cyclic, which implies it's the Miquel point of both quadrilaterals, since
is already on
and
. In particular, this means that
has a Simson line with respect to both complete quadrilaterals, which will be useful soon.
We now prove Sondat's Theorem for Paralogic Triangles. Let
be the orthocenter of
and
the orthocenter of
. The theorem states that the line
bisects
. Note that when
is a Simson line, then
, and the theorem is the classic statement that the Simson line bisects
.
Let
be the Steiner line of
with respect to
, which is just the Simson line scaled up by a factor of
at
. By the lemma above, we see that it passes through
. But since
is the Miquel point, the line
also passes through the reflection of
in
, which we'll call
. Thus, line
is
.
Let
be the corresponding Steiner line of
with respect to
. We see that
is sent to
under the spiral similarity at
, so
By the above, we also have
. This implies that
. Thus,
is the perpendicular bisector of
, which implies that it passes through the center
, which is the midpoint of
, as desired. This completes the proof. 
![[asy]
unitsize(1.5inches);
pair A,B,C,D,E,F,X,Y,Z,P,Q,H,HH,R;
A=dir(105);
B=dir(210);
C=dir(330);
F=(0.3)*A+(0.7)*B;
E=(3/5)*A+(2/5)*C;
D=extension(E,F,B,C);
X=2*circumcenter(A,E,F)-A;
Y=2*circumcenter(B,D,F)-B;
Z=2*circumcenter(C,D,E)-C;
P=extension(A,X,B,Y);
Q=2*foot(P,circumcenter(A,B,C),circumcenter(X,Y,Z))-P;
H=orthocenter(A,B,C);
HH=orthocenter(X,Y,Z);
R=2*foot(Q,D,E)-Q;
draw(A--B--C--cycle,linewidth(1.5)+red);
draw(D--B);
draw(X--Y--Z--cycle,linewidth(1.5)+orange);
draw(X--E,dotted);
draw(circumcircle(A,B,C));
draw(D--F--E,blue);
draw(A--X--P,green);
draw(P--B--Y,green);
draw(Z--P--C,green);
draw(circumcircle(X,Y,Z));
dot("$A$",A,dir(A));
dot("$B$",B,dir(230));
dot("$C$",C,dir(C));
dot("$F$",F,1.5*dir(105));
dot("$E$",E,dir(0));
dot("$D$",D,dir(180));
dot("$X$",X,dir(-10));
dot("$Y$",Y,dir(160));
dot("$Z$",Z,dir(200));
dot("$P$",P,1.3*dir(270));
dot("$Q$",Q,1.3*dir(110));
dot("$H$",H,1.3*dir(0));
dot("$H'$",HH,1.3*dir(180));
dot("$R$",R,1.3*dir(-30));
[/asy]](http://latex.artofproblemsolving.com/8/4/2/84257186b6f8aa72f72176e4e9c53b6fadd2c304.png)
Consider














It's clear that




![\[\angle BCP=\angle DCZ=\angle DEZ=\angle FEX=\angle FAX=\angle BAP,\]](http://latex.artofproblemsolving.com/b/9/8/b984b441625c2c151d95668b5059e6a289069cc1.png)




It turns out that















The point









We now prove Sondat's Theorem for Paralogic Triangles. Let









Let













Let














Projective Gauss Line
by yayups, May 13, 2019, 6:43 AM
The inspiration (and much of the content) of this post came from a discussion I was having recently with some people.
In this post, we'll outline a projective viewpoint on the Gauss Line, but first, let's give the standard presentation.
![[asy]
unitsize(1inches);
pair A=1.1*dir(110);
pair B=1.5*dir(65);
pair C=dir(-50);
pair D=0.7*dir(180);
pair P=extension(A,D,B,C);
pair Q=extension(A,B,C,D);
pair H=orthocenter(B,C,Q);
pair U=foot(B,Q,C);
pair V=foot(C,B,Q);
pair W=foot(Q,B,C);
pair X=IP(CP((A+C)/2,A),CP((D+B)/2,B));
pair Y=OP(CP((A+C)/2,A),CP((D+B)/2,B));
dot("$A$",A,dir(A));
dot("$B$",B,dir(B));
dot("$C$",C,dir(C));
dot("$D$",D,dir(230));
dot("$P$",P,dir(90));
dot("$Q$",Q,dir(180));
dot("$H$",H,1.9*dir(150));
dot("$U$",U,dir(260));
dot("$V$",V,dir(90));
dot("$W$",W,dir(-20));
draw(C--P);
draw(C--Q);
draw(D--P);
draw(B--Q);
draw(B--U,dashed+red);
draw(C--V,dashed+red);
draw(Q--W,dashed+red);
draw(CP((A+C)/2,A),dotted+blue);
draw(CP((D+B)/2,B),dotted+blue);
draw(CP((P+Q)/2,P),dotted+blue);
draw(X--Y,green);
[/asy]](//latex.artofproblemsolving.com/9/3/e/93ea3115a9832a30c9f31c63318bd43790ce0f9a.png)
The proof goes as follows. Let
be the orthocenter of
, and let
be the orthic triangle of
. Note that the power of
with respect to
is
, the power with respect to
is
, and the power with respect to
is
. These are all equal, so
has equal power with respect to
,
, and
. We may repeat a similar argument for the orthocenters of
,
, and
, and we see then that
,
, and
are coaxial. Thus, their centers are collinear, and this proves the existence of the Gauss line.
There are two things to check. Firstly, by taking a limit of appropriate thin ellipses inscribed in
, one that approaches segment
, one for segment
, and one for
, we see that these midpoints are on this locus. However, the main thing we need to check is that this locus is a line. Note that projectively, the center of a conic is the pole of the line at infinity with respect to the conic.
So the problem reduces to the following:
It will be much easier to prove the dual statement, which we can get by letting
,
,
, and
.
We'll prove this version. Let
be the line tangent to
at
.
Let
be some random conic passing through
. By Desargue's involution theorem with line
, we see that there is some fixed involution (dependent only on
) that swaps the two points
. By construction,
is a fixed point of this involution, so there must be a second fixed point, call it
. We see that
and
are fixed by this involution, so this involution must be harmonic conjugation in
. Thus,
so
lies on the polar of
with respect to
, as desired (note that
is only dependent on
and
). 
This concludes the projective classification of the Gauss Line. Note that this then trivializes the theorem that the incenter of
(if it exists) lies on the Gauss Line.
In this post, we'll outline a projective viewpoint on the Gauss Line, but first, let's give the standard presentation.
Gauss Line wrote:
For a complete quadrilateral
(
and
), the midpoints of
,
and
are all collinear.






![[asy]
unitsize(1inches);
pair A=1.1*dir(110);
pair B=1.5*dir(65);
pair C=dir(-50);
pair D=0.7*dir(180);
pair P=extension(A,D,B,C);
pair Q=extension(A,B,C,D);
pair H=orthocenter(B,C,Q);
pair U=foot(B,Q,C);
pair V=foot(C,B,Q);
pair W=foot(Q,B,C);
pair X=IP(CP((A+C)/2,A),CP((D+B)/2,B));
pair Y=OP(CP((A+C)/2,A),CP((D+B)/2,B));
dot("$A$",A,dir(A));
dot("$B$",B,dir(B));
dot("$C$",C,dir(C));
dot("$D$",D,dir(230));
dot("$P$",P,dir(90));
dot("$Q$",Q,dir(180));
dot("$H$",H,1.9*dir(150));
dot("$U$",U,dir(260));
dot("$V$",V,dir(90));
dot("$W$",W,dir(-20));
draw(C--P);
draw(C--Q);
draw(D--P);
draw(B--Q);
draw(B--U,dashed+red);
draw(C--V,dashed+red);
draw(Q--W,dashed+red);
draw(CP((A+C)/2,A),dotted+blue);
draw(CP((D+B)/2,B),dotted+blue);
draw(CP((P+Q)/2,P),dotted+blue);
draw(X--Y,green);
[/asy]](http://latex.artofproblemsolving.com/9/3/e/93ea3115a9832a30c9f31c63318bd43790ce0f9a.png)
The proof goes as follows. Let





















Projective Gauss Line wrote:
The Gauss line is the locus of the centers of all inscribed conics to
.

There are two things to check. Firstly, by taking a limit of appropriate thin ellipses inscribed in




So the problem reduces to the following:
Rephrased Problem v1 wrote:
Fix some line
, and consider the family of conics
inscribed in
. The locus of the pole of
in
is a line.









Rephrased Problem v2 wrote:
Fix some point
, and consider the family of conics
that pass through some fixed points
. The polar of
in
passes through a fixed point.





We'll prove this version. Let



Let










![\[(LL';UV)=-1\]](http://latex.artofproblemsolving.com/5/b/f/5bf10e704961b4ff8dd339906bc1ee05f1031fb9.png)







This concludes the projective classification of the Gauss Line. Note that this then trivializes the theorem that the incenter of

This post has been edited 2 times. Last edited by yayups, Aug 4, 2019, 6:59 AM
Projective Bary
by yayups, May 11, 2019, 6:06 AM
This solution shows a nontrivial usage of the projective nature of barycentric coordinates.
![[asy]
/* Geogebra to Asymptote conversion, documentation at artofproblemsolving.com/Wiki go to User:Azjps/geogebra */
unitsize(0.2inches);
import graph; size(0cm);
real labelscalefactor = 0.5; /* changes label-to-point distance */
pen dps = linewidth(0.7) + fontsize(10); defaultpen(dps); /* default pen style */
pen dotstyle = black; /* point style */
real xmin = -10, xmax = 14, ymin = -22.62, ymax = 13.46; /* image dimensions */
pen zzttqq = rgb(0.6,0.2,0);
draw((0.26,7.92)--(-1.52,-2.18)--(9.54,-2.2)--cycle, linewidth(2) + zzttqq);
/* draw figures */
draw((0.26,7.92)--(-1.52,-2.18), linewidth(2) + zzttqq);
draw((-1.52,-2.18)--(9.54,-2.2), linewidth(2) + zzttqq);
draw((9.54,-2.2)--(0.26,7.92), linewidth(2) + zzttqq);
draw(circle((4.017668907550992,2.050905875698934), 6.968955550057753), linewidth(2));
draw(circle((2.278232171040842,1.001527516890443), 3.1883907164579206), linewidth(2));
draw((5.8094995867931365,8.78556876819826)--(0.07556713698788581,3.306761151157378), linewidth(2));
draw((0.07556713698788581,3.306761151157378)--(-2.791521751662705,0.5672401366735647), linewidth(2));
draw((-2.791521751662705,0.5672401366735647)--(-5.658856324457237,-2.1725156305163527), linewidth(2));
draw((-5.658856324457237,-2.1725156305163527)--(-1.52,-2.18), linewidth(2));
draw((-2.791521751662705,0.5672401366735647)--(5.215750162966643,-4.8142920649363115), linewidth(2));
draw((5.215750162966643,-4.8142920649363115)--(5.8094995867931365,8.78556876819826), linewidth(2));
/* dots and labels */
dot((0.26,7.92),dotstyle);
label("$A$", (0.34,8.12), N * labelscalefactor);
dot((-1.52,-2.18),dotstyle);
label("$B$", (-1.44,-1.98), NE * labelscalefactor);
dot((9.54,-2.2),dotstyle);
label("$C$", (9.62,-2), E *3* labelscalefactor);
dot((2.2724665553022745,-2.1868579865376176),linewidth(4pt) + dotstyle);
label("$D$", (2.36,-2.02), NE * labelscalefactor);
dot((5.215750162966643,-4.8142920649363115),linewidth(4pt) + dotstyle);
label("$A'$", (5.3,-4.66), S * 4*labelscalefactor);
dot((-2.791521751662705,0.5672401366735647),linewidth(4pt) + dotstyle);
label("$B'$", (-3.48,0.62), NW*0.01 * labelscalefactor);
dot((5.8094995867931365,8.78556876819826),linewidth(4pt) + dotstyle);
label("$C'$", (5.88,8.94), NE * labelscalefactor);
dot((-5.658856324457237,-2.1725156305163527),linewidth(4pt) + dotstyle);
label("$T$", (-6.02,-1.92), NE * labelscalefactor);
dot((0.07556713698788581,3.306761151157378),linewidth(4pt) + dotstyle);
label("$X$", (0.02,3.66), N * labelscalefactor);
clip((xmin,ymin)--(xmin,ymax)--(xmax,ymax)--(xmax,ymin)--cycle);
/* end of picture */
[/asy]](//latex.artofproblemsolving.com/9/2/6/9265c88f4a81d8aca6c5ad3b7c4cd51da79b1f5f.png)
Firstly, the problem is clearly just asking us to show that
is tangent to
. Letting
, we see that this is equivalent to
, but
, so all we need to show is that
We'll actually just compute the barycentric coordinates of
and verify this explicitly. However, finding
and
is really messy, and this doesn't seem tractable immediately. However, the beauty of this solution is getting a simple way to find the coordinates of
.
Let
be a variable point on
, and define the composite map
given by
where
is the
inverse of
and
and
are tangent to
with
. It's not easy to see that this map
is projective but its true.
Why?
Now, we check that
so
, and similarly
. Also,
but uh-oh, what's
? The fix is that we actually have the tangent to
at some point infinitesimally close to
, which then intersects
really close to
, so we have
.
Now, use projective coordinates for
given by the last two coordinates of the barycentric coordinates of a point on
. The map
looks like some matrix
that is considered the same under scaling. We see that
and
map to themselves, so
. The point
is given by
, and it maps to
, so the matrix is
Now, we see that
since
and
are
inverses (the incircle gets sent to the mixtilinear excircle), so
so
. The rest is a boring bash which we do below.
Homogenizing, we have
,
,
, and
. By taking simply the first coordinate, we get a coordinate system on
that preserves the lengths in the original setup, so
and
, and
Evidently, we have
, as desired. As we showed before, this means
, so
is tangent to
, which is tangent to
at
, so
is tangent to
. 
USAMO Shortlist 2014, 110 Geometery Problems #77 wrote:
Let
be a triangle with circumcircle
and incircle
. Let
be the mixtilinear excircle touch point, and let
and
be tangent to
with
. Let
be the tangency point of
with
which exists by Poncelet's Porism. Show that
is tangent to
.













![[asy]
/* Geogebra to Asymptote conversion, documentation at artofproblemsolving.com/Wiki go to User:Azjps/geogebra */
unitsize(0.2inches);
import graph; size(0cm);
real labelscalefactor = 0.5; /* changes label-to-point distance */
pen dps = linewidth(0.7) + fontsize(10); defaultpen(dps); /* default pen style */
pen dotstyle = black; /* point style */
real xmin = -10, xmax = 14, ymin = -22.62, ymax = 13.46; /* image dimensions */
pen zzttqq = rgb(0.6,0.2,0);
draw((0.26,7.92)--(-1.52,-2.18)--(9.54,-2.2)--cycle, linewidth(2) + zzttqq);
/* draw figures */
draw((0.26,7.92)--(-1.52,-2.18), linewidth(2) + zzttqq);
draw((-1.52,-2.18)--(9.54,-2.2), linewidth(2) + zzttqq);
draw((9.54,-2.2)--(0.26,7.92), linewidth(2) + zzttqq);
draw(circle((4.017668907550992,2.050905875698934), 6.968955550057753), linewidth(2));
draw(circle((2.278232171040842,1.001527516890443), 3.1883907164579206), linewidth(2));
draw((5.8094995867931365,8.78556876819826)--(0.07556713698788581,3.306761151157378), linewidth(2));
draw((0.07556713698788581,3.306761151157378)--(-2.791521751662705,0.5672401366735647), linewidth(2));
draw((-2.791521751662705,0.5672401366735647)--(-5.658856324457237,-2.1725156305163527), linewidth(2));
draw((-5.658856324457237,-2.1725156305163527)--(-1.52,-2.18), linewidth(2));
draw((-2.791521751662705,0.5672401366735647)--(5.215750162966643,-4.8142920649363115), linewidth(2));
draw((5.215750162966643,-4.8142920649363115)--(5.8094995867931365,8.78556876819826), linewidth(2));
/* dots and labels */
dot((0.26,7.92),dotstyle);
label("$A$", (0.34,8.12), N * labelscalefactor);
dot((-1.52,-2.18),dotstyle);
label("$B$", (-1.44,-1.98), NE * labelscalefactor);
dot((9.54,-2.2),dotstyle);
label("$C$", (9.62,-2), E *3* labelscalefactor);
dot((2.2724665553022745,-2.1868579865376176),linewidth(4pt) + dotstyle);
label("$D$", (2.36,-2.02), NE * labelscalefactor);
dot((5.215750162966643,-4.8142920649363115),linewidth(4pt) + dotstyle);
label("$A'$", (5.3,-4.66), S * 4*labelscalefactor);
dot((-2.791521751662705,0.5672401366735647),linewidth(4pt) + dotstyle);
label("$B'$", (-3.48,0.62), NW*0.01 * labelscalefactor);
dot((5.8094995867931365,8.78556876819826),linewidth(4pt) + dotstyle);
label("$C'$", (5.88,8.94), NE * labelscalefactor);
dot((-5.658856324457237,-2.1725156305163527),linewidth(4pt) + dotstyle);
label("$T$", (-6.02,-1.92), NE * labelscalefactor);
dot((0.07556713698788581,3.306761151157378),linewidth(4pt) + dotstyle);
label("$X$", (0.02,3.66), N * labelscalefactor);
clip((xmin,ymin)--(xmin,ymax)--(xmax,ymax)--(xmax,ymin)--cycle);
/* end of picture */
[/asy]](http://latex.artofproblemsolving.com/9/2/6/9265c88f4a81d8aca6c5ad3b7c4cd51da79b1f5f.png)
Firstly, the problem is clearly just asking us to show that





![\[TB\cdot TC=TD^2.\]](http://latex.artofproblemsolving.com/2/d/8/2d8ca7873f2e762e57966529e2fb19f52d6cb0da.png)




Let



![\[P\mapsto P'\mapsto P_1P_2\cap BC\]](http://latex.artofproblemsolving.com/3/b/0/3b0a1bc34ad2accec60eebb852a299c9b94cb17e.png)








Why?
We're going to discuss why this map is projective. Let
be the tangency point of
to
. We have this Poncelet map from
given by
, and it's projective since it's bijective and can be constructed with ruler/compass. Now, the point
is the pole of
, which is projective, so
is projective. It's important to note that
is not projective since it doesn't pass through a fixed point, but since we're intersecting it with a line also tangent to
, it works out by using pole/polar.










Now, we check that
![\[B\mapsto C\mapsto AB\mapsto B,\]](http://latex.artofproblemsolving.com/b/6/9/b696c5b4d3d959c53822f3791ee22cc6e6fb3996.png)


![\[\infty_{BC}\mapsto A\mapsto BC,\]](http://latex.artofproblemsolving.com/7/0/5/705c34756f4cd1195dadbcd751763ef30a458a08.png)






Now, use projective coordinates for



![\[\begin{pmatrix}w & x\\ y& z\end{pmatrix}\]](http://latex.artofproblemsolving.com/9/b/9/9b96ab082094cdb16b53fba91dd088fb597fc6c8.png)






![\[\phi\equiv \begin{pmatrix}s-c & 0\\ 0 & -(s-b)\end{pmatrix}.\]](http://latex.artofproblemsolving.com/6/8/b/68bd66cd7a38db2d1345380b628b589a960ff750.png)




![\[T=\begin{pmatrix}s-c & 0\\ 0 & -(s-b)\end{pmatrix}\begin{pmatrix}s-c \\ s-b\end{pmatrix}=\begin{pmatrix}(s-c)^2 \\ -(s-b)^2\end{pmatrix},\]](http://latex.artofproblemsolving.com/1/1/2/112091d2ae6406785da1a9ec592ac043583852a5.png)

Homogenizing, we have





![\[TB=\frac{(s-c)^2}{(b-c)a}-1=\frac{(s-b)^2}{(b-c)a}\]](http://latex.artofproblemsolving.com/d/7/0/d702095640ed1b38129962cce048d18b092682dd.png)

![\[TD=\frac{(s-c)^2}{(b-c)a}-\frac{s-c}{a}=\frac{s-c}{a}\cdot\frac{s-b}{b-c}=\frac{(s-b)(s-c)}{a(b-c)}.\]](http://latex.artofproblemsolving.com/e/c/f/ecf4625af98721f9c35d194c39dcd73b6482258b.png)









This post has been edited 3 times. Last edited by yayups, May 11, 2019, 10:35 PM
Combinatorial Games
by yayups, Feb 14, 2019, 2:58 AM
Here are 2 nice combinatorial games problems that I solved recently. They are very similar as explained in the remark below.
Solution
Solution
Remark, also a spoiler
USAMO 1999/5 wrote:
The Y2K Game is played on a
grid as follows. Two players in turn write either an S or an O in an empty square. The first player who produces three consecutive boxes that spell SOS wins. If all boxes are filled without producing SOS then the game is a draw. Prove that the second player has a winning strategy

View the board position as a string of the characters S,O,-. Call the players Alice and Bob.
We prove a series of lemmas to prove that Bob will win if both players play optimally.
Lemma 1: Bob can make sure that there is some turn of his where the board has a substring S--S.
Proof of Lemma 1: If Alice opens with placing an S somewhere, then Bob simply places another S to get the substring S--S somewhere. We're not done yet since it is now Alice's turn. But if she plays in one of the two blanks in the S--S, it is easy to see that Bob wins, so Alice must play outside the --. Therefore, it is Bob's turn, and the substring S--S exists.
Now, assume she instead opens with playing an O somewhere. Bob should then place an S far from the O and the edges of the board (fifty squares from the O and the edges should suffice). Alice won't play so that Bob can complete an SOS, since she is playing optimally, so she plays something else. Now, on Bob's second move, he has two ways to make the S--S, but at most one of them could lead to a winning position for Alice because of her second move. So he just plays the one that won't lead to a winning position for Alice, so Alice's third turn is not a win, so Bob's third turn arrives with the substring S--S existing somewhere on the board.
An important consequence of this lemma is that the game will now not draw - eventually someone must play in the blanks in S--S, and then they lose.
Lemma 2: Once Bob has S--S somewhere, Bob can have the ends of the board to be filled on one of his turns.
Proof of Lemma 2: Suppose Bob has S--S and it is his turn, and WLOG Bob can't immedeatly win (else game over). If an edge square is empty, by putting an O on the edge, Bob adds no winning positions, so Alice makes a move that doesn't finish, so its Bob's turn again. Thus, Bob can fill up the edges.
Lemma 3: If Bob can't win on a turn, then he can play so that he will be around to play another turn.
Proof of Lemma 3: Suppose there is a substring of the form -O or O-. Certainly if Bob adds an O to make them OO, then he could not have added any new winning substring (a winning substring is SO-, S-S, or -OS). Thus, WLOG assume that there are no substrings of the form -O or O-.
Thus, the board looks like
where
is a string of S and Os that starts and ends with S (except maybe
and
that could start and end with O respectively), and has no substring of the form SOS, and
for all
. If any of the
, then Bob should simply put an S in the first - in
. It is easy to see that this adds no winning substrings, so it suffices to examine the case
. But this means that the number of squares placed so far is
, which is even, so it can't even be Bob's turn! Therefore, we are done. 
Lemma 1 implies that there is no draw, and lemma 3 implies that Bob won't lose, so Bob must win.
QED
We prove a series of lemmas to prove that Bob will win if both players play optimally.
Lemma 1: Bob can make sure that there is some turn of his where the board has a substring S--S.
Proof of Lemma 1: If Alice opens with placing an S somewhere, then Bob simply places another S to get the substring S--S somewhere. We're not done yet since it is now Alice's turn. But if she plays in one of the two blanks in the S--S, it is easy to see that Bob wins, so Alice must play outside the --. Therefore, it is Bob's turn, and the substring S--S exists.
Now, assume she instead opens with playing an O somewhere. Bob should then place an S far from the O and the edges of the board (fifty squares from the O and the edges should suffice). Alice won't play so that Bob can complete an SOS, since she is playing optimally, so she plays something else. Now, on Bob's second move, he has two ways to make the S--S, but at most one of them could lead to a winning position for Alice because of her second move. So he just plays the one that won't lead to a winning position for Alice, so Alice's third turn is not a win, so Bob's third turn arrives with the substring S--S existing somewhere on the board.

An important consequence of this lemma is that the game will now not draw - eventually someone must play in the blanks in S--S, and then they lose.
Lemma 2: Once Bob has S--S somewhere, Bob can have the ends of the board to be filled on one of his turns.
Proof of Lemma 2: Suppose Bob has S--S and it is his turn, and WLOG Bob can't immedeatly win (else game over). If an edge square is empty, by putting an O on the edge, Bob adds no winning positions, so Alice makes a move that doesn't finish, so its Bob's turn again. Thus, Bob can fill up the edges.

Lemma 3: If Bob can't win on a turn, then he can play so that he will be around to play another turn.
Proof of Lemma 3: Suppose there is a substring of the form -O or O-. Certainly if Bob adds an O to make them OO, then he could not have added any new winning substring (a winning substring is SO-, S-S, or -OS). Thus, WLOG assume that there are no substrings of the form -O or O-.
Thus, the board looks like
![\[X_1(-)^{a_1}X_2(-)^{a_2}\cdots (-)^{a_n}X_{n+1}\]](http://latex.artofproblemsolving.com/1/c/9/1c91fc301309e35f9cb54f782289ec427d8afc3d.png)










Lemma 1 implies that there is no draw, and lemma 3 implies that Bob won't lose, so Bob must win.
QED
ISL 2015 C4 wrote:
Let
be a positive integer. Two players
and
play a game in which they take turns choosing positive integers
. The rules of the game are:
(i) A player cannot choose a number that has been chosen by either player on any previous turn.
(ii) A player cannot choose a number consecutive to any of those the player has already chosen on any previous turn.
(iii) The game is a draw if all numbers have been chosen; otherwise the player who cannot choose a number anymore loses the game.
The player
takes the first turn. Determine the outcome of the game, assuming that both players use optimal strategies.
Proposed by Finland




(i) A player cannot choose a number that has been chosen by either player on any previous turn.
(ii) A player cannot choose a number consecutive to any of those the player has already chosen on any previous turn.
(iii) The game is a draw if all numbers have been chosen; otherwise the player who cannot choose a number anymore loses the game.
The player

Proposed by Finland
We claim that
wins except for
where the game draws.
The right way to think about this problem is the following. Players
and
are placing tokens on an
grid (
's tokens and
's tokens are distinguishable). A player cannot play a token next to a token they played before. A player loses if they cannot play on their turn, and its a draw if all the squares get filled up.
WLOG, assume
doesn't play square
(if she does, then flip the board). We have the following wondrous lemma.
Lemma: If
plays square
on his first turn, then she is guaranteed to not lose (it may draw however).
Proof of Lemma: On the
th turn of player
, there are
tokens on the board, and there is one
token all the way to the left of everything. These
tokens create
so called "holes" - basically the region between two consecutive
s. Since the
s can't be next to each other, these holes have nonzero size (they might be filled with
s). But there are only
s potentially in these holes (because one
is at
- the point was to safeguard this
), so there is some hole with no
s in it. Therefore,
has some place to play, so
doesn't lose. Note that
does not have to play in that hole - all we're saying is that worst case,
always has somewhere to play. 
From this we can resolve almost all cases for
.
Claim 1: If
is odd, then
wins.
Proof of Claim 1: Assume that
does not win. Again, we assume
does not play
.
should now play
, and by the lemma, the game must draw. But this means
played all
squares, so to not have any adjacent ones, she must have played on all the odds. But
played on an odd, so we have the desired contradiction. 
Claim 2: If
is even, then
wins. The idea is the same as Claim 1, but the details are a bit more involved.
If
doesn't open with playing
or
, then
plays on
or
to match the parity of
's move. By the same argument as in claim 1,
wins. Therefore, WLOG
plays
. Now,
should play
. By the lemma, the game draws (we're going by contradiction I suppose). If
plays something less than
, then the squares
are accessible to
, and one of them has got to be even.
should play that and then win by the same logic as in claim 1.
Therefore,
s second move must be
. Now, if
, then the squares
have an even number, and
can play there, and win by the same argument as in claim 1. 
It suffices to resolve the cases
. We see that
are obviously draws. It is not too much harder to do a brute force check (check the relatively small tree of game outcomes) that
are also draws. So we're done. 


The right way to think about this problem is the following. Players





WLOG, assume


Lemma: If


Proof of Lemma: On the





















From this we can resolve almost all cases for

Claim 1: If


Proof of Claim 1: Assume that









Claim 2: If


If

















Therefore,






It suffices to resolve the cases




Remark, also a spoiler
The idea in both goes like the following:
Player 2 sets up stuff in the beginning. Then we show that player 2 can always live another turn without losing. Finally, the set up in the beginning plus parity shows that Player 1 actually can't win, so player 2 must win.
Note that in the C4 its even better - player 2 can make a move to in fact not lose no matter what she does. In the USAMO problem, player 2 still has to be careful to live another turn.
Player 2 sets up stuff in the beginning. Then we show that player 2 can always live another turn without losing. Finally, the set up in the beginning plus parity shows that Player 1 actually can't win, so player 2 must win.
Note that in the C4 its even better - player 2 can make a move to in fact not lose no matter what she does. In the USAMO problem, player 2 still has to be careful to live another turn.
Mathematicians are Cleverer than Physicists
by yayups, Jan 11, 2019, 7:08 AM
USA TST 2015/3 wrote:
]A physicist encounters
atoms called usamons. Each usamon either has one electron or zero electrons, and the physicist can't tell the difference. The physicist's only tool is a diode. The physicist may connect the diode from any usamon
to any other usamon
. (This connection is directed.) When she does so, if usamon
has an electron and usamon
does not, then the electron jumps from
to
. In any other case, nothing happens. In addition, the physicist cannot tell whether an electron jumps during any given step. The physicist's goal is to isolate two usamons that she is sure are currently in the same state. Is there any series of diode usage that makes this possible?
Proposed by Linus Hamilton







Proposed by Linus Hamilton
Label the usamons




Lemma: If there exists

![\[x_{\sigma(1)}\le x_{\sigma(2)}\le\cdots\le x_{\sigma(n)},\]](http://latex.artofproblemsolving.com/c/2/4/c24c75707cd96cd07b2b9c9963fc187bfaeb0165.png)

Proof of Lemma: If










![\[x_{\sigma'(1)}\le x_{\sigma'(2)}\le\cdots\le x_{\sigma'(n)}.\]](http://latex.artofproblemsolving.com/3/a/0/3a08e40c19c55d09a2b9f61ec0d757601cc4f97f.png)

This actually implies that if the physicist has information
![\[x_{\sigma(1)}\le x_{\sigma(2)}\le\cdots\le x_{\sigma(n)},\]](http://latex.artofproblemsolving.com/c/2/4/c24c75707cd96cd07b2b9c9963fc187bfaeb0165.png)
![\[x_{\sigma'(1)}\le x_{\sigma'(2)}\le\cdots\le x_{\sigma'(n)}.\]](http://latex.artofproblemsolving.com/3/a/0/3a08e40c19c55d09a2b9f61ec0d757601cc4f97f.png)





But the physicist starts with no information, but we showed that even if she knew the

![\[x_{\sigma(1)}\le x_{\sigma(2)}\le\cdots\le x_{\sigma(n)},\]](http://latex.artofproblemsolving.com/c/2/4/c24c75707cd96cd07b2b9c9963fc187bfaeb0165.png)

Remarks Originally, I was trying to show that the physicist's knowledge at any time was a poset on the




Moving points tutorial
by yayups, Jan 5, 2019, 9:50 AM
I think a lot of people will really like this. (original link here https://artofproblemsolving.com/community/c6h1763036)
Hi All,
I've been getting many requests lately to explain this technique, so here is a short writeup which explains the method of projective maps from scratch. Any suggestions are welcome, and I hope it helps!
Basic familarity with projective geometry is assumed. Mainly just good intuition with cross ratios is required.
Let
be a conic, a line, or a pencil of lines through a fixed point (note that this last one is often ommited from the definition, but I find it to be quite important). These objects have a structure of cross ratio built into them, specifically, if
, then we can talk about
(note that if
is a pencil of lines, then
are lines). The definition of a projective map is now very simple.
Definition A projective map
from
to
where these are either conics, lines, or a pencil of lines is a function that preserves cross ratio. In particular, if
, then
And that's all!
The reason this is useful is the following theorem.
Theorem If
are projective, then
if
and
coincide on three different input values.
Proof: Suppose
,
,
. Then,
The important equation is
. Since the cross ratio is bijective, we have
, as desired. 
When solving problems, if we can phrase the problem as in the theorem, then actually we only have to check the problem for 3 cases! Before doing examples, let's cover some basic transformations that are projective maps. Your intuition with cross ratios is going to be really helpful here. Firstly, note that the composition of two projective maps is projective, and the inverse of a projective map is projective (Exercise: Prove this from the definition!). Here is a list I could come up with for projective maps (not exhaustive by any means).
You might be wondering where projecting from a line to a line fits in. It turns out this is actually a composition of two of the maps we listed above. In particular, if
is a point and
are lines, then we first map
using the first map, then map
using the inverse of the first map. Their composition is the classic perspectivity map. It's time to do some examples!
![[asy] size(8cm);
pair A = dir(129); pair B = dir(220); pair C = dir(320); filldraw(A--B--C--cycle, invisible, black);
pair M = 0.5*A + 0.5*B; pair N = 0.5*A + 0.5*C; pair O = circumcenter(A,M,N); pair Q = rotate(-90,A)*O; pair X = 4.2*Q - 3.2*A; pair Y = intersectionpoints(circle(X,abs(X-A)),circumcircle(A,M,N))[0]; pair Z = 0.5*A + 0.5*Y;
filldraw(circumcircle(C,Y,N), invisible, blue+opacity(0.5));
draw(A--X,red); pair S = circumcenter(C, N, Y); pair T = foot(S, B, C); pair D = 2*T - C; pair P = extension(S, midpoint(N--C), X, N); draw(N--X);
dot("$A$", A, dir(A)); dot("$B$", B, dir(B)); dot("$C$", C, dir(0)); dot("$M$", M, dir(135)); dot("$N$", N, dir(80)); dot("$X$", X, dir(X)); dot("$D$", D, dir(225)); [/asy]](//latex.artofproblemsolving.com/1/6/6/1663ff6c1db80a29582615c8e7cf1f07a223ee24.png)
Diagram made by v_Enhance. Let
be the tangent at
to
and let
. Let
be the map from
to
given by
. Note that this means
(directed angles).
So consider the map from
to
given by rotation by angle
. This is clearly projective. We have the map from
by
, and we have
by
. Therefore, composing all these projective maps in the order
we have that
is a projective map that maps
.
Similarly, we define a map
that sends
. We have again that its projective. We want to show
, so by the theorem, we just have to check this for three choices of
. This is now a geometry problem that is left to the reader!
Note: This proof is written much shorter like this. Let
. Then,
is projective, so
is projective. Similarly if
, then
is projective. So it suffices to check for three values of
.
The next example will have the proof again written nice and short like the above.
![[asy]
import olympiad;
import cse5;
size(8cm);
defaultpen(fontsize(9pt));
pair A = dir(110);
dot("$A$", A, dir(A));
pair B = dir(210);
dot("$B$", B, dir(B));
pair C = dir(330);
dot("$C$", C, dir(C));
pair I = incenter(A, B, C);
dot("$I$", I, dir(45));
pair O = origin;
pair D = extension(A, I, O, B+C);
dot("$D$", D, dir(225));
pair E = dir(310);
dot("$E$", E, dir(E));
pair F_1 = B*C/E;
pair F = extension(B, C, A, F_1);
dot("$F$", F, dir(F));
pair G = midpoint(I--F);
dot("$G$", G, dir(-15));
pair K = extension(E, I, D, G);
dot("$K$", K, dir(K));
pair I_A = 2*D-I;
pair P = extension(D, K, A, F);
pair Z = extension(I, P, I_A, F);
draw(A--B--C--cycle, red);
draw(unitcircle, blue);
draw(A--E--K--D--cycle, green);
draw(I--F);
[/asy]](//latex.artofproblemsolving.com/0/4/6/046adea104842e9f6d362caf7c8735e7a89e8679.png)
Diagram from v_Enhance. We will animate
on
, which means that we have mappings generated by varying
. Let
and
. We have that
is projective by the third item on our list.
We'll now show that
is projective. Note that
is projective since
is a reflection which clearly preserves cross ratio. Also,
is projective since it is a homothety at
with factor
(here the codomain is the scaling of
by
at
). Thus,
is projective. Projecting through
, we have that
is projective, so
is projective, as deisred.
Therefore, to show
, it suffices to check three values of
. It is now a geometry problem (an easy one at that) to check the problem for
(possible hint: fact 5).
Hi All,
I've been getting many requests lately to explain this technique, so here is a short writeup which explains the method of projective maps from scratch. Any suggestions are welcome, and I hope it helps!
Basic familarity with projective geometry is assumed. Mainly just good intuition with cross ratios is required.
Let





Definition A projective map




![\[(AB;CD) = (f(A)f(B);f(C)f(D)).\]](http://latex.artofproblemsolving.com/e/e/f/eef3e6921bf3e50a0478e5605bee11f297ab1906.png)
The reason this is useful is the following theorem.
Theorem If




Proof: Suppose



![\[(AB;CD)=(f(A)f(B);f(C)f(D))=(g(A)g(B);g(C)g(D))=(f(A)g(B);f(C)g(D)).\]](http://latex.artofproblemsolving.com/4/3/7/43702b598276b26d7f8611e252df0b17cb61c3a7.png)



When solving problems, if we can phrase the problem as in the theorem, then actually we only have to check the problem for 3 cases! Before doing examples, let's cover some basic transformations that are projective maps. Your intuition with cross ratios is going to be really helpful here. Firstly, note that the composition of two projective maps is projective, and the inverse of a projective map is projective (Exercise: Prove this from the definition!). Here is a list I could come up with for projective maps (not exhaustive by any means).
- Given a line
and a point
, the map from
to
(the pencil of lines through
) given by
- Given a conic
and a point
on the conic, the map from
to
given by
- Given a conic
and any point
, the map from
to
by
- This one's a little off beat, but still useful. Given two clines
and
, any inversion (or Mobius transform for that matter) that sends
to
is projective. This is because of the famous fact that inversion preserves cross ratio.
You might be wondering where projecting from a line to a line fits in. It turns out this is actually a composition of two of the maps we listed above. In particular, if




USA Winter TST for IMO 2019 Problem 1 wrote:
Let
be a triangle and let
and
denote the midpoints of
and
, respectively. Let
be a point such that
is tangent to the circumcircle of triangle
. Denote by
the circle through
and
tangent to
, and by
the circle through
and
tangent to
. Show that
and
intersect on line
.
Merlijn Staps



















Merlijn Staps
![[asy] size(8cm);
pair A = dir(129); pair B = dir(220); pair C = dir(320); filldraw(A--B--C--cycle, invisible, black);
pair M = 0.5*A + 0.5*B; pair N = 0.5*A + 0.5*C; pair O = circumcenter(A,M,N); pair Q = rotate(-90,A)*O; pair X = 4.2*Q - 3.2*A; pair Y = intersectionpoints(circle(X,abs(X-A)),circumcircle(A,M,N))[0]; pair Z = 0.5*A + 0.5*Y;
filldraw(circumcircle(C,Y,N), invisible, blue+opacity(0.5));
draw(A--X,red); pair S = circumcenter(C, N, Y); pair T = foot(S, B, C); pair D = 2*T - C; pair P = extension(S, midpoint(N--C), X, N); draw(N--X);
dot("$A$", A, dir(A)); dot("$B$", B, dir(B)); dot("$C$", C, dir(0)); dot("$M$", M, dir(135)); dot("$N$", N, dir(80)); dot("$X$", X, dir(X)); dot("$D$", D, dir(225)); [/asy]](http://latex.artofproblemsolving.com/1/6/6/1663ff6c1db80a29582615c8e7cf1f07a223ee24.png)
Diagram made by v_Enhance. Let









So consider the map from







![\[\ell_1\to\mathcal{C}_N\to\mathcal{C}_N\to\ell_2,\]](http://latex.artofproblemsolving.com/2/d/b/2db26114caa7a4f9bfbc2ab8e648312be613996f.png)


Similarly, we define a map




Note: This proof is written much shorter like this. Let

![\[X\mapsto NX\mapsto ND\mapsto D\]](http://latex.artofproblemsolving.com/6/6/3/663e83070f354db6cc3506378d2f63795acc3a1d.png)




The next example will have the proof again written nice and short like the above.
IMO 2010/2 wrote:
Given a triangle
, with
as its incenter and
as its circumcircle,
intersects
again at
. Let
be a point on the arc
, and
a point on the segment
, such that
. If
is the midpoint of
, prove that the meeting point of the lines
and
lies on
.
















![[asy]
import olympiad;
import cse5;
size(8cm);
defaultpen(fontsize(9pt));
pair A = dir(110);
dot("$A$", A, dir(A));
pair B = dir(210);
dot("$B$", B, dir(B));
pair C = dir(330);
dot("$C$", C, dir(C));
pair I = incenter(A, B, C);
dot("$I$", I, dir(45));
pair O = origin;
pair D = extension(A, I, O, B+C);
dot("$D$", D, dir(225));
pair E = dir(310);
dot("$E$", E, dir(E));
pair F_1 = B*C/E;
pair F = extension(B, C, A, F_1);
dot("$F$", F, dir(F));
pair G = midpoint(I--F);
dot("$G$", G, dir(-15));
pair K = extension(E, I, D, G);
dot("$K$", K, dir(K));
pair I_A = 2*D-I;
pair P = extension(D, K, A, F);
pair Z = extension(I, P, I_A, F);
draw(A--B--C--cycle, red);
draw(unitcircle, blue);
draw(A--E--K--D--cycle, green);
draw(I--F);
[/asy]](http://latex.artofproblemsolving.com/0/4/6/046adea104842e9f6d362caf7c8735e7a89e8679.png)
Diagram from v_Enhance. We will animate






We'll now show that













Therefore, to show



This post has been edited 1 time. Last edited by yayups, Mar 27, 2019, 11:40 PM
Archives



















Shouts
Submit
93 shouts
Tags
About Owner
- Posts: 1614
- Joined: Apr 17, 2013
Blog Stats
- Blog created: Jun 26, 2017
- Total entries: 45
- Total visits: 37903
- Total comments: 64
Search Blog